« first day (32 days earlier)      last day (542 days later) » 
00:00 - 15:0015:00 - 00:00

12:03 AM
0
Q: Show that clu(B) = empty set in a metric space M

Daniellet A be contained in M such that A is nonempty but clu(A) is the empty set. Assume A does not equal M. Let y be an element of M-A and let B=A union {y}. show that clu(B) is the empty set

0
Q: Clarification of Erdos-Szekeres theorem, monotonic subsequence length

Ghost_StarkI know the theorem states that if you have $n^2 + 1$ real numbers in a sequence then you are guaranteed to have an increasing or decreasing monotonic subsequence of length $n+1$. I'm trying to understand the proof of this theorem using the pigeon hole principle. I was following this answer here,...

0
Q: proof diagonalizing Euclidean Domain Matric

AUAPlease help, i'm learning about introduction of abstrac algebra and now i'm facing a question related to Ecludian domain. Sadly up until now i coudnt find any clue how i should start proofing this question, i do hope anyone can help. Known that R Euclidean Domain, A is a matrix nxn with it's ent...

 
12:22 AM
0
Q: Matrix variables

IronCanTacoWhat I need to do is find values of t where matrix equals 0. Apparently, those two numbers are -1 and 0, but how do I calculate them? Do I search them with determinant? enter image description here

Short title. Short question. Matrix variables
0
Q: Pidgeonhole Principle proof involving sets of integers

MroogProve that every set of $2k$ integers has elements of opposite parity differing by at least $2k-1$ or elements of equal parity differing by at least $4k-2$.

0
Q: Darboux Integral

AnonymousLet $f$ be defined on $[0,1]$ by $f(x):=1$ if $x \not= 1$ and $f(1):=0$, Show that the Darboux Integral exist and find its value. I know I want my partition to be $P_\epsilon := (0, 1-\epsilon/2, 1+\epsilon/2,2)$ I'm having trouble defining my Upper sum and Lower sum

Short title. Darboux Integral
0
Q: prove $a≡_nb$ then $a^k≡_nb^k$

Noah DengIf $a≡_nb$ for some $n≥2n$, then $n|a−b|$, thus $n|(a−b)(a+b)=a^2−b^2$. Thus $a^2≡_nb^2$. I am confused where to add the induction to prove that for any $a^k≡_nb^k$

Short question. Tagged proof-verification. prove $a≡_nb$ then $a^k≡_nb^k$
0
Q: continuous function on the reals with dense image in the complex plane

user1566114Let $f:\mathbb{R}\rightarrow\mathbb{C}$ be continuous with a dense image. How does one show that the preimage under $f$ of an open ball in $\mathbb{C}$ is unbounded?

0
Q: Tangent Question: Find the Y-int?

user295692Let C be defined by y=f(x).Suppose there's a tangent at (4, 3). The tangent equation is y=2x+5. Let C be the curve y=xf(x)/1+x^2 Find the y-int of tangent to C at the point with x-condition that =4 For this question what I did was just use y=mx+b, make it y=(2)(4)+5 which equals 13. So woul...

Welcome to Math.SE, user295692. Words such as question are uninformative in titles. Please edit the title so that it better describes the specifics of your question. Do not hesitate to make it longer or include a formula if needed. This site uses MathJax formatting of formulas. More tips here. (autocomment)Normal Human 21 secs ago
0
Q: Cosinus hyperbolicus calculation

QuestionnaireCould you please check me? cosh (ln(sqrt5)) = ? cosh (x) = e^x - e^-x/2 So it should be like this e^ln(sqrt5)) - e^-(ln(sqrt5))/2 sqrt5-1/sqrt5/2 sqrt5/2+1/(2*sqrt5) 3/sqrt5 Thanks.

This site uses MathJax formatting of formulas. More tips here. (from a bot)Normal Human 21 secs ago
0
Q: Poisson bracket computation

user1620696Consider $L_i$ the $i$-th component of angular momentum defined by $$L_i = \sum_{r,s}x^rp_s\epsilon_{rsk},$$ being $x^r$ the $r$-th component of position and $p_s$ the $s$-th component of momentum, though of as coordinate functions on the phase space $T^\ast\mathbb{R}^3$. I'm trying to show tha...

0
Q: Is the subset a subspace?

friedmanfanboyI'm given a subset \begin{equation} S = \{f \in C[-1,1]: f^{-1}[\{0\}]=\{0\}\} \end{equation} and am supposed to determine if it is a subspace of $C[-1,1]$. Now, I don't think there exists a function that is continuous on $[-1,1]$ whose inverse determined at $0$ is equal to $0$, and therefore th...

0
Q: Matrix decompisition to toeplitz and diagonal

M AtrixWhat is a method that I can decompose a full rank matrix, say H, into a series of toeplitz (T) and diagonal (D) matrices? That is how to do: $H=T_1P_1T_2P_2...T_NP_N$

0
Q: Trigonometric Identity Question

Dunja ElezIf $x$ is in the interval $[0, {\pi \over 2}]$ and $y$ is in the interval $[{\pi \over 2}, \pi]$ and $tan x={4 \over 3}$ and $csc y={13 \over 5}$, evaluate $$sin(x+y)$$ The final answer is supposed to be ${-33 \over 65}$ I started by graphing $cscy={13 \over 5}$ on a cartesian plane. Because o...

Words such as question do not add information to titles. Please edit the title so that it better describes the specifics of your question. Do not hesitate to make it longer or include a formula if needed. More tips here. (from a bot)Normal Human 21 secs ago
0
Q: Normal Disatribution

felipe alvesI'm tryng to do a exercise about normal distribution. But I think the statement is wrong,because de the value of Z is very big. μ=2000 hours standard deviation=40 Exercise: Probability between 1450 hours and 1580 hours Recipe: Z=(X-μ)/standard deviation

Short title. Normal Disatribution
0
Q: simple functions

user295699function {(2,4)(-8,9)} is a set of number for f(x) f(x) -> px + q 2p + q = 4 -8 + q = 9 10p = -5 p = -2 I only get p ,but I wasnt sure whether correct or not . Help me

Short title. simple functions
0
Q: Using the residue theorem

user295703Is it possible to evaluate $$\int_{-\infty}^\infty \frac{x^2}{(x^2+1)^2} \, dx $$ using the residue theorem, as opposed to Calc 1 methods?

Short title. Short question. Using the residue theorem
0
Q: Discrete Mathematics Proving Question - Please help

JohnnyI need help with this proof. I am stuck on this question and don't know how to do it: Prove that: $$ \forall n \in \mathbb{N}, \sum\limits_{i=2}^{2^n} \frac{1}{i} \geq \frac{n}{2}$$

Words such as question, please, help are uninformative in titles. Please edit the title so that it better describes the specifics of your question. Do not hesitate to make it longer or include a formula if needed. More tips here. (autocomment)Normal Human 21 secs ago
0
Q: Doubt in Natural Deduction

Ricardo BrandaoI have a doubt to prove by natural deduction of this: $\vdash\forall x \forall y \varphi (x,y) \rightarrow \forall x \varphi(x,x)$ First of all, I took $\vdash\forall x \forall y \varphi (x,y)$ as a hipothesis And eliminate $\forall y$, to deduce: $\forall x \varphi(x,x)$. My question is: May I c...

0
Q: i need help understanding the proof that the borromean rings do not exist.

Raphadoes somebody know about the Borromean rings or knots in general and can summarize the key steps of the proof? i don't get it.

Words such as doubt are uninformative in titles. Please edit the title so that it better describes the specifics of your question. Do not hesitate to make it longer or include a formula if needed. More tips here. (autocomment)Normal Human 21 secs ago
0
Q: Inequalities involving integrals please help

e_glaudSo using the fact that root 3 + x/4 is less than or equal to root (x+3) which is less than or equal to 2 on the interval [0,1], I have to prove that root 3 plus 1/20 is less than or equal to the integral of root (x^4 + 3) dx from 0 to 1, which is less than or equal to 2. How do I go about doing...

Welcome to Math.SE, e_glaud. Words such as please, help do not add information to titles. Please edit the title so that it better describes the specifics of your question. Do not hesitate to make it longer or include a formula if needed. This site uses MathJax formatting of formulas. More tips here. (autocomment)Normal Human 21 secs ago
0
Q: nonabelian group of order pq

joeysuppose G is a nonabelian group of order pq, where p and q are both prime number. prove that if the nontriaval element a belongs to G ,then =C(a).

Short title. Short question. nonabelian group of order pq
 
1:06 AM
0
Q: Summation that appears to be zero

erdekhayserHere is the expression in question: $$\lim_{n\to\infty} \sum_{k=1}^{n} \Big[2 + \frac{3}{n}k\Big]^2 \Big(\frac{3}{n}\Big)$$ The first thing I noticed was this: $$\frac{3}{n}$$ I figured that as n approaches infinity, this value would approach zero, so the value for the entire expression would...

Question contains please. Summation that appears to be zero
0
Q: Show that $f: G \to H $ is a homomorphism.

Shojul000This is my first encounter with homomorphisms and I'd like to have my proof verified. Question: Let $G = (\mathbb{Z}, +)$ and $H = \{6^{n} \mid n \in \mathbb{Z} \}$. Define $f: G \to H$ by $f(x) = 6^{x}$. Show this is a homomorphism. Attempt: We want to show $f(a + b) = f(a) \cdot f(b)$ for all...

0
Q: Group action on Finite Field

AngelSuppose $F=\mathbb{F}_{p^n}$ is a degree-$n$ extension of $\mathbb{F}_p$. My questions concerns the action of the multiplicative group $(\mathbb{F}_p)^{\times}$ on $F$ by left multiplication. then we can write $F=\mathbb{F}_p[x]/\langle \pi\rangle$ for some monic irreducible polynomial $\pi\in \m...

0
Q: Increasing question function?

user295692I do not understand this question AT ALL. f(x)=lnx/x for xe(0, infinity) a) Show that e is the only continuous number of f(x) b) Show that f'(x) > 0 for all xe(0, e) and f'(x) <0 for all xe(e, infinity) c) Explain why part b implies that f(e) is an absolute maximum value of f on (0, infinity) d...

Words such as question are uninformative in titles. Please edit the title so that it better describes the specifics of your question. Do not hesitate to make it longer or include a formula if needed. More tips here. (from a bot)Normal Human 21 secs ago
0
Q: Basic conceptual questions about orthogonality in Linear Algebra,

Lebron JamesI have the Gram-Schmidt algorithm memorized, so that I can always compute an orthonormal basis, when I need it (on pen and paper, I don't studying mathematical / scientific computing ... yet). Could I think of the Gram-Schmidt algorithm geometrically in the two / three-dimensional cases? What d...

0
Q: Catering problem. How many will eat wheat sugar and are vegetarians (combinatorics)

NaokiYou are catering the awards banquet. One hundred will be attending. 8 do not eat wheat,sugar or meat. 68 eat wheat, 48 eat wheat but are vegetarians, 18 eat wheat but not sugar,50 eat wheat and sugar, 20 people eat wheat and are not vegetarians, 20 eat sugar and not wheat. How many eat wheat and ...

0
Q: Understand free and bound variable associations in Lambda Calculus

ryanI understand that free variables in Lambda calculus are those that are not bound to a specific metavariable inside of an abstraction, while bound variables are the direct opposite. The idea that confuses me is that of parenthetical placement inside of an expression/series of expressions. express...

Welcome to Math.SE, ryan. Questions tend to get more attention when they have a tag for a broad area of mathematics relevant to the question. Some of these tags might fit. (autocomment)Normal Human 21 secs ago
0
Q: Quotient Rings example

user270494I have a field = $F_2$ and a polynomial $x^2 + x + 1$ over that field. I understand that since the polynomial is degree 2, it has no root in $F_2$, so it is irreducible. But why is $F_2[X]/(x^2+x+1)$ a field with 4 elements? (0, 1, x, x+1) Also when I do the multiplication table of the elements...

0
Q: Problem regarding differentiability

ArupI am looking for the solution of the following problem, Let $f: \mathbb{R} \rightarrow \mathbb{R}$ a twice differentiable function with $f(0) =2, f^{\prime}(0)=2$ and $f(1)=1.$ Then there is $x \in (0,1)$ such that $f(x)f^{\prime}(x)+f^{\prime \prime}(x)=0.$ I think we have to use mean value t...

Short title. Title contains problem. Problem regarding differentiability
0
Q: Expectation of S?

user3904534There are $n$ marbles and $r$ boxes. One at a time, each marble is selected and randomly (uniformly) placed in one of the $r$ boxes. Let $S$ be the number of empty boxes. Compute $E(S)$ and $Var(S )$. Here is my work: Let $X$ = the box is empty This gives me: \begin{equation} X_i = \begin{cas...

Short title. Expectation of S?
0
Q: Very simple system of two first order ordinary differential equations. (3.1-1)

Jules MansonI am not certain what to do here. Please assist by setting up the first two or three steps of this very simple system of first order ordinary differential equations. My differential equations textbook states to use the "elimination method" to crack this. We may also solve by the matrix method whe...

0
Q: How to simplify this Sum Of Products equation

manicmailmanI am having trouble simplifying this equation to fit the truth table. A'B'C'D'+ A'B'C'D + A'B'CD' + A'B'CD + A'BC'D + A'BCD' + AB'C'D + AB'CD' + AB'CD + ABCD Any Help would be awesome, thanks

0
Q: How do I evaluate $\displaystyle \int_{0}^{\infty} \frac{1}{\sqrt{2 \pi s}} e^{-z^{2}/2s} \cdot \frac{1}{2}e^{-s/2} \, ds$?

M.S.EHow do you evaluate: $$\displaystyle \int_{0}^{\infty} \frac{1}{\sqrt{2 \pi s}} e^{-z^{2}/2s} \cdot \frac{1}{2}e^{-s/2} \, ds=?$$

Welcome to Math.SE, manicmailman. This site uses MathJax formatting of formulas. More tips here. (from a bot)Normal Human 21 secs ago
0
Q: How many 10-bit binary sequences begin and end with the same bit?

user295723Can you please teach me how to solve this? There are other similar questions that I have, but I don't understand how to solve them. Another question is: How many 10-bit binary sequences contain an odd number of 1s?

 
2:03 AM
1
Q: Calculus- Solve the initial value problem dy/dx

Jon RoyHey people of Stack exchange, I missed one class this week and missed the lecture on Anti derivatives, I am just wondering how to solve this question Solve the Initial Value problem. $\frac {dy} {dx} = x^3$, $y(0) = 7$ $y= ?$

0
Q: Suppose a vector x is orthogonal to both vectors y and z. Prove that x is orthogonal to any vector in span{y, z

BNSlugSorry, don't have many ideas on how to start this one. Is it because of some relation between y and z?

Consider adding a tag for a broader subject area to which the question belongs. Some of these tags might fit. (from a bot)Normal Human 21 secs ago
0
Q: How to solve integral 0 to 1 of (ln(x+1))/(x^2+1) dx

user295727I'm looking for a way to solve the above equation only using U-substitution without any trig. Again the problem is: integral 0 to 1 of (ln(x+1))/(x^2+1) dx

 
2:22 AM
0
Q: Irrational flow on torus

BrutusI have an interesting dynamical systems problem that has had me stumped for a few hours now, so I'm hoping I can get some help. The problem is concerned with flows on the torus. The model is given by $\dot {\theta}_1=\omega_1$ and $\dot {\theta}_2=\omega_2$, where $\theta_{1,2}$ are the phases of...

0
Q: A function of Class $C^2$

CjugWe have f as a function of $\Bbb R \to \Bbb R$ and is of Class $C^2$. $f(0)=1/2$, $f'(0)=1$, $f''(x) \le 1 $ $\forall x \in[-1,1]$. Show that over [-1,1] we have: $$|f(c+h)-f(c)-f'(c)h|\le \frac{h^2}{2}$$, for $c, c+h \in [-1,1]$. I know that f is of Class $C^2$ if all the partial derivatives up...

Short title. Tagged differential-equations but mentions "partial". A function of Class $C^2$
0
Q: How would I solve this absolute value inequality?

AlexI am just wondering how I'd 'solve the following inequalities' 1) |x+1| < 4 2) |3x+2| ≥ 2-x Thanks

0
Q: Discrete math evaluate

Jonathan LeeQuestion: Evaluate $$\sum_{k=0}^n {2k\choose k}{2n-2k \choose n-k}$$ Hint: use the fact that $$(1-4x)^{-1/2} = \sum_{n\ge0} {2n \choose n}x^n$$ For some reason the hint has me more lost than the problem but I'm sure it is included for a reason. My gut feeling is to approach this with a binomial s...

 
0
Q: Losing a bounty because OP doesn't check in

charlestoncrabbIs there a protocol for/anything you can do about a bounty expiring simply because the OP has not revisited the question? I recently posted my first answer to a bounty question, for which I was pretty excited since the bounty q's are generally quite involved. I was proud of my answer and it was ...

 
0
Q: Proving of this trigonometric identity

Nitratecot B/(csc B - 1) + cot B/(csc B + 1) = 2 sec B What I've done: (cos B/sin B)/(1/sin B) - 1 + (cos B/sin B)/(1/sin B) + 1 (cos B/sin B)/(1 - sin B)/sin B + (cos B/sin B)/(1 + sin B)/sin B (cos B(1 - sin B))/sin^2 B + (cos B(1 + sin B))/sin^2 B (cos B - cosBsinB + cosB + cosBsinB) / sin^2 B ...

This site uses MathJax formatting of formulas. More tips here. (autocomment)Normal Human 21 secs ago
0
Q: Differentiation of $e^{sin x}$

tatanHow do I solve ? $$\frac{d}{dx}e^{sinx}$$ A very detailed explanation is expected as I am a beginner.Thanks for any help.

0
Q: Discrete cross-correlation: How to do cross correlation in FFT with segmented windows?

Nhân LêI have a discrete signal: $x=[i_1, i_2, ..., i_n]$ I would like to do the cross correlation between sub-groups of (k number of) consecutive number of $x$, for example: $xcorr([i_1, i_2, i_3], [i_2, i_3, i_4])$ What is the most efficient way to do this, computationally? I have been thinking of ...

Welcome to Math.SE, Nhân Lê. Questions tend to get more attention when they have a tag for a broad area of mathematics relevant to the question. Some of these tags might fit. (autocomment)Normal Human 21 secs ago
0
Q: Minimizing DFA equivalence

VaroagOriginal question: Consider carrying out the state minimization algorithm. We'll let Partition 0 be the name of the initial partition of the states into sets N = {Q0, Q1, Q2, Q3, Q4} and F = {Q5}. The next is Partition 1, the next Partition 2, and so forth. Based on this graph this question w...

0
Q: Some doubt in conditional expectation

SoshaLet $X$ and $Y$ be two random elements in a Hilbert space. Then I have seen in a paper to use the following $E[<X,Y>] = E[<E[X|Y], Y>]$ Here $E[X|Y]$ is also an element of the Hilbert space

Words such as doubt do not add information to titles. Please edit the title so that it better describes the specifics of your question. Do not hesitate to make it longer or include a formula if needed. More tips here. (from a bot)Normal Human 20 secs ago
0
Q: If C = (F^-1) AF and also C = (G^-1)BG, what matrix M gives B = (M^-1)AM?

Ayechan_SanSecond part of the question: If C is similar to A and also to B then __ (Not sure if this is correct but I answer: A is similar to B?) Please help me with the first part of the question, I'm very confused.

0
Q: Can Any one give me some hints?

Gillian CheungCan Any one give me some hints? Please dont give me the answer i still want to left some room for myself to think .May anyone give me some advice so that i can solve the questions Questions: Le K be a nonempty compact subset of R. Prove that there exists a bounded continuous function f :R->R s....

Question contains please. Can Any one give me some hints?
0
Q: Mathematical Induction Proof, need help on how to explain this one statement.

PaulUse Mathematical Induction to prove that for [; n>=1 ;], that $ b_n=(1/2)((3^n)+1) $ Solution: Basic case: For [; n = 1 ;] $$ b_1 = 2 = 1/2((3^1)+1) $$ Assume that for some k $$ b_k = 1/2(3^k + 1) $$ Then $$ b_{k+1} = 3b_k - 1 $$ $$ = 3(1/2)(3^k + 1) - 1 $$ ...

0
Q: Prove using induction the 2 color theorem proof

Noah DengA box with 4 closed sides has $n$ straight lines going all the way across it. The base case is of course only 1 line which separates the box into 2 colors, black and white. How can I show using induction that for any $n$ lines, the box can still be colored with only black and white. I am famil...

0
Q: Dimension of subspace of commuting matrices

MaxI was hoping for some confirmation of a proof to the following preliminary exam question: Fix an $n \times n$ matrix $A$ with entries in an algebraically closed field $k$. Let $C$ be the space of $n \times n$ matrices over $k$ that commute with $A$. Observe that $C$ is a vector space over ...

0
Q: Help understanding expected values and probabilities

unsure_ee_studentAt a party, ten men throw their hats into the center of a room. The hats are mixed up and each man randomly selects one. (Where we assume that if a hat is chosen, it can't be chosen again). A) Define $X_i$ , such that for i = 1,2,..,10, appropriately such that $X = \sum_i^{10} X_i$ = X So if ...

Welcome to Math.SE, unsure_ee_student. Words such as help are uninformative in titles. Please edit the title so that it better describes the specifics of your question. Do not hesitate to make it longer or include a formula if needed. More tips here. (from a bot)Normal Human 21 secs ago
0
Q: Prove Darboux integrable

AnonymousProve that if $g(x):=0$ for $0\le x\le1/2$and $g(x):=1$ for $1/2\lt x\le1$ then the Darboux Integral of $g$ on $[0,1]$ is equal to $1/2$. My answer: Lets define a partition $P_\epsilon =(0,1/2,1/2+\epsilon,1)$ thne the upper sum $U(g,P_\epsilon)=0(1/2-0)+1(1/2+\epsilon-1/2)+1(1-(1/2+\epsilon)=0+...

Short title. Tagged proof-verification. Prove Darboux integrable
0
Q: If A = diag(1, 3) and B = diag(3, 1) show that A and B are similar (find an M).

Ayechan_SanPlease show me steps how you would solve it. I'm really lost and detailed explanation would help me learn and understand it better.

0
Q: Convexity of log-determinant

user23658I'm having a difficult time determining if the following function is convex: $$f(X) = \log {\rm det}(X^T A X), $$ where $A \in \mathbb{R}^{r \times r}$ is a symmetric positive definite matrix and $X \in \mathbb{R}^{u \times r}$ with $r < u$ and $X'X = I$. I've worked on find the second derivati...

0
Q: 1 meet your new counterpart, i?

Gareth ComptonSo my question is is the fourth root of 1 = i? Where i^4 = 1? or if it is still 1 nonetheless?

0
Q: Competing Probability

KennethJSuppose I have two cumulative probability distributions: $P_a(x_a)$ and $P_b(x_b)$. How do I combine these two distributions to find the combined probability that one random variable is lower than the other. As in, how do I find $P(x_a < x_b)$? I'm familiar enough with statistics to be able to c...

0
Q: Inequality regarding the logarithm

guachoI have found the inequality $$ x\leq cy+\log(1/c)(x-y)\log(x/y), $$ for all $x,y>0$ and $c>1$. Why is this inequality true?

0
Q: STATS Math Question: Pearson Correlation /Item prediction

Shivam GuptaI am working on a computer science assignment. I am suppose to create a function which calculates this equation !http://imgur.com/a/9rIDY Now the 2nd image shows the calculation for the table1. How would the calculation change for the table in pic 1.

Welcome to Math.SE, Shivam Gupta. Words such as question are uninformative in titles. Please edit the title so that it better describes the specifics of your question. Do not hesitate to make it longer or include a formula if needed. More tips here. (from a bot)Normal Human 21 secs ago
0
Q: sqrt{2} if and only if X and Y are standard normal

user119758Consider F is a closed set in $\mathbb R$, is it true that F $\cap$ $\mathbb Q$ is dense in F? I failed to show it...$\sqrt{2}$

 
4:06 AM
0
Q: I made an edit to a question adding additional details I learned of after implementing my aceppted answer, but a high rep user has reverted my edit?

user89The question: I installed a program by getting its source code, and then running `sudo make install`; how to make `apt-get` know about it? The edit that was removed: (revision history here) Edit for posterity: As the top answer suggests, checkinstall should be used whenever possible instead ...

 
0
Q: Let $Q:=[0,2] \times [0,2]$ ; then how to evaluate ${\int\int}_Q\lfloor x+y\rfloor dxdy$ ?

Saun DevLet $Q:=[0,2] \times [0,2]$ ; then how to evaluate ${\int\int}_Q\lfloor x+y\rfloor dxdy$ ? where $\lfloor . \rfloor$ denotes the greatest integer function . Please help . Thanks in advance

Short question. Question contains please. [Let $Q:=[0,2] \times [0,2]$ ; then how to evaluate ${\int\int}_Q\lfloor x+y\rfloor dxdy$ ?](math.stackexchange.com/q/1559233)
0
Q: Redefining vector addition for subset of vector space

dissemLet $W$ be the following subset of $\mathbb{R}^{3}$: $ W = $ { $(x,y,z) : x + 2y + z = 2$ }. Define a new vector addition rule that makes $W$ closed under vector addition.

0
Q: Why the equation can be equal to $(1-p)^k$?

user277100$P(X>k)$ =$\displaystyle \sum_{i\ge k+1}p(1-p)^{i-1}$ =$(1-p)^k$ I don't understand why it can be equal to $(1-p)^k$?

0
Q: Simplification with squared fractions

Vinicius L. BeserraI´m trying to simplify the following fractions 1/(z-d/2)^2-1/(z+d/2)^2. And I got the the answer (z+d/2)^2/(z-d/2)^2-(z-d/2)^2/(z+d/2)^2. But I not sure what I can do with the denominators.

This site uses MathJax formatting of formulas. More tips here. (from a bot)Normal Human 21 secs ago
0
Q: Prove or disprove a monotonicity property of a certain convex optimization problem

JobstLet $R = (r_{ij})$ be an $n\times k$ real matrix with only positive entries, and consider the convex optimization problem $\max f(x) = \sum_{i=1}^n \log \sum_{j=1}^k r_{ij} x_j$ such that $\sum_{j=1}^k x_j = 1$ and $x_j\ge 0$ for all $j=1...k$. Assume that $R$ is such that this problem has a un...

 
Nice wall of crap.
2
 
4:29 AM
0
Q: What is Gaussian Environment

kneelb4darth The perceptron bears a certain relationship to a classical pattern classifier known as the Bayes classifier. When the environment is Gaussian, the Bayes classifier reduces to a linear classifier.This is the same form taken by the perceptron. What is a Gaussian Environment referred to in ...

Welcome to Math.SE, kneelb4darth. Questions tend to get more attention when they have a tag for a broad area of mathematics relevant to the question. Some of these tags might fit. (autocomment)Normal Human 21 secs ago
0
Q: Fun Q5: Square Tower

numberphileSquare building blocks of side length $a$ are used to build a tower. Each layer has to contain a certain amount of blocks to hold the tower standing. Let $F_n$ be the number of blocks required in the $n^{th}$ layer from the top. Then $F_n = F_{n-1} + F_{n-2}$ where $F_1 = F_2 = 1$ A tower of $...

Short title. Fun Q5: Square Tower
0
Q: Volume between a cylinder and 2 plane

Ryan CusackThis is a question that I had on one of my midterms. I completely bombed this one, I still have no idea how to solve it. Any help would be appreciated :) -- Find the volume of the solid region bounded above by the plane z = x, on the sides by the cylinder x^2+y^2=y, and below by the xy-plane. ...

Welcome to Math.SE, Ryan Cusack. This site uses MathJax formatting of formulas. More tips here. (from a bot)Normal Human 21 secs ago
0
Q: Double Angle identity???

Dunja ElezThe question asks to fully solve for $$(sin{\pi \over 8}+cos{\pi \over 8})^2$$ My question is, is this a double angle formula? And if so, how would I go about to solve it? I interpreted it this way; $$(sin{\pi \over 8}+cos{\pi \over 8})^2$$ $$=2sin{\pi \over 4}+(1-2sin{\pi \over 4})$$ Have...

Short title. Title contains ??. Tagged algebraic-identities. Double Angle identity???
0
Q: Show that f is diagonalizable

user295760The question is: Suppose that $f : \mathbb{R}^2 \rightarrow \mathbb{R}^2$ is linear, and that $f^{2015} = I$. Show that $f$ is diagonalizable. [hint: If f is a multiple of the identity, it’s easy. Otherwise, it suffices to prove that f has two distinct eigenvalues.] My proof so far: Let $\vec{...

0
Q: Proof for ceiling property?

user295761Prove that floor(2x)=floor(x)+floor(x+(1/2)). enter image description here

Short title. Short question. Proof for ceiling property?
 
0
Q: Announcer Badge Question

GambleNerdI have read over the auto-populated questions and other documentation or help I can find and I have some questions specific to the Announce Badge (see below) and exactly where it's allowed to be used to be applied. I'f there is some obvious resource that goes over my questions about this topic, ...

 
0
Q: A function that is integrable over the closure of an open set but not over the set itself

DkficI'm trying to find a bounded function $f$ and an open set $A$ such that $f$ is integrable over $\overline{A}$ but not over $A$. Is there such a function and set?

0
Q: A question about systems of linear equations?

user262291I was wondering whether a homogeneous system of linear equations with more variables than equations always has a solution other than the trivial solution. If so do you mind explaining why.

Words such as question do not add information to titles. Please edit the title so that it better describes the specifics of your question. Do not hesitate to make it longer or include a formula if needed. More tips here. (from a bot)Normal Human 20 secs ago
0
Q: Matrix Determinants Equivalence

John Snoe The left-hand side becomes A*I_n - B*0_n,n = A, correct? How can A = det(A) just from the information given?

Short title. Short question. Matrix Determinants Equivalence
0
Q: Properties of Estimators

emkaSuppose I have the following pdf $f(y;\theta)=\frac{1}{\theta^2}ye^{-y/\theta}$ and I am told that $E(Y)=2\theta$ and $Var(Y)=2\theta^2$. Is $\frac{1}{2}\bar{Y}$ an unbiased estimator of $\theta$? Is $\frac{1}{2}\bar{Y}$ an efficient estimator of $\theta$? First, is it unbiased. Conside...

0
Q: Prove that if f:A1 -> A2 and g:B1 -> B2 are bijections then h:A1xB1 -> A2xB2, h(a,b)=(f(a),g(b)) is a bijection.

Stacy (a) Prove that if f:A1 -> A2 and g:B1 -> B2 are bijections then h:A1xB1 -> A2xB2, h(a,b)=(f(a),g(b)) is a bijection. Please refer to image of problem for clearer formatting.

0
Q: Figure out number of possible equations counting

Brandon TurpyHow many solutions are there to the equation x1 + x2 + x3 + x4 + x5 + x6 = 25 where each xi is a non-negative integer, 3 ≤ x1 ≤ 10, 2 ≤ x2 ≤ 7 and x3 ≥ 5 I have been able to do all my counting problems but this one. I can not find an equation for max number per variety. I know that I will subtr...

This site uses MathJax formatting of formulas. More tips here. (from a bot)Normal Human 21 secs ago
 
5:03 AM
0
Q: Probability of grabbing pencils and pens from two bins

user1634494The problem is states that there are two bins, Bin #1 which has 3 pens and 7 pencils; and Bin #2 with 8 pens and 4 pencils. (a) A bin is chosen at random and an object is drawn. What is the probability that it is a pencil? I am thinking that this is just P(pencil | bin #1) * P(bin #1) + P(penci...

0
Q: Having problem with dtermining the path over which a line integral is to be evaluated

Saun DevLet $C$ be the curve of intersection of $z=xy ; x^2+y^2=1$ traversed once in a direction that appears counterclockwise when viewed from High above the $xy$-plane ; then how to evaluate $\int_Cydx+zdy+xdz$ ? I have found a parametrization of the curve which is $\Big(\cos t ,\sin t , \dfrac {\sin ...

This site uses MathJax formatting of formulas. More tips here. (autocomment)Normal Human 21 secs ago
0
Q: Parabolic arclength question

JShoeConsidering a parabola of the form $y=ax^2+bx$ I was curious what relation $a$ and $b$ would need to have in order for the arclength of the parabola between its zeroes to be twice the distance between its zeroes. To simplify the calculus, I set the integral from the first zero to the vertex equ...

Words such as question do not add information to titles. Please edit the title so that it better describes the specifics of your question. Do not hesitate to make it longer or include a formula if needed. More tips here. (autocomment)Normal Human 21 secs ago
0
Q: basic math question to obtain fair competition between two teams

SanathConsider a class with 4 students having min goals as{1, 3, 4, 5} and max goals as{2, 5, 8, 6} fine the best way to divide the class in such a way that the match is competitive i.e. the maximum difference between goals scored by the team is minimized My solution: after taking the average of the m...

0
Q: About Trivial homomorphism

Helenif I want to prove that " there does not exist any homomorphism f : → Can I just prove there only exist f is trivial homomorphism? Thanks

Short title. Short question. About Trivial homomorphism
0
Q: system differential equation

MikeConsider the two-dimensional system x' = -x^4 + 5µx^(2)-4µ^(2) y'=-y I found that there are four critical points for this system. but i do not know how the how draw the phase portraits for various values of µ and draw the bifurcation diagram.

0
Q: Perfect numbers.

QwertyWhile reading an article about perfect numbers I noticed 1 thing. All of them are even. I tried to find some logic behind this unusual property, because I thought that the logic might be too naive to be stated, so they haven't stated. But unfortunately , I couldn't get any success. Can someone he...

0
Q: Problem in Topology related to continuous functions

Razor1692Let R-Q be the subspace of R with the usual metric.Is there a function, f:R-Q-->R-Q such that f is continuous and f does not have a fixed point? I get that if we consider the metric R every continuous function from R to R has a fixed point.But we consider R-Q ,firstly i cant think of a continuo...

Questions tend to get more attention when they have a tag for a broad area of mathematics relevant to the question. Some of these tags might fit. (autocomment)Normal Human 21 secs ago
0
Q: Reflection of curve.

TIWARI Let the curve C be the mirror image of the parabola $y^2= 4 x$ with respect to the line $x+y+4=0$. If A and B are the points of intersection of C with the line $y=-5$, then the distance between A and B is. Answer =4. I have solved it as, first translating the coordinate to (-2,-2), then re...

Short title. Reflection of curve.
0
Q: Normal subgroup definition confusion

abstractGoneIf $H <G$, then $H$ is called normal if $gHg^{-1} \subset H$ for all $g\in G$. Why is the definition not $gHg^{-1} = H$? We are not assuming commutivity for $G$. Take $gH = Hg$, multiply both sides by $g^{-1}$ and we get $gHg^{-1} = Hgg^{-1}$. Why is $Hgg^{-1} \neq H$ always?

 
5:33 AM
0
Q: SQL Parsing Dynamically in Stored Procedure

user3430726I've to write a stored Procedure and I was wondering if anyone can help me to parse input AttributeList from below SQL. Technically I need to break AttributeList down to different attributes comma separated such as atr1, atr2, .. atrN etc and pass them in query group by clause. Query - SELE...

 
0
Q: Discrete Math- Hard Problem

ConnerGive an example of an injective function f : N → (0, 1), and prove that your function is injective.

Short title. Title contains hard, problem. Short question. Discrete Math- Hard Problem
0
Q: Determine whether the function $f\colon\mathbb{Z}_{3}\oplus\mathbb{Z}_4\to\mathbb{Z}_{12}$ given by $f([a],[b])=[4a+9b]$ is a ring isomorphism.

cheDetermine whether the function $f\colon\mathbb{Z}_{3}\oplus\mathbb{Z}_4\to\mathbb{Z}_{12}$ given by $f([a],[b])=[4a+9b]$ is a ring isomorphism. How does I even solve this? please help me

Question contains please. [Determine whether the function $f\colon\mathbb{Z}_{3}\oplus\mathbb{Z}_4\to\mathbb{Z}_{12}$ given by $f([a],[b])=[4a+9b]$ is a ring isomorphism.](math.stackexchange.com/q/1559332)
0
Q: Half Angle Identities ???

Dunja ElezIf $\sin{x \over 2}={2 \over 3}$, $0 \le x \le {\pi \over 2}$, evaluate $$\cos x$$ I'm not sure how to start. I have graphed $\sin{x \over 2}={2 \over 3}$ on a cartesian plane but I am stuck. Am I supposed to set $\cos x$ to a half identity?

Short title. Title contains ??. Tagged algebraic-identities. Half Angle Identities ???
0
Q: Differential Equations 3333

Mikeshow this differential equation x''+sin(x)= 0 is not structurally stable ? I do not know how to covert it to a system of ode.

Short title. Short question. Differential Equations 3333
 
5:52 AM
0
Q: User adding irrelevant tags

Ben S.There have been a couple days now when at around the same time, a certain user has been going through old posts marked with the 'probability' tag and adding editing them to add irrelevant tags, most commonly changing 'probability' to 'probability-theory' when it is not necessary and adding the ta...

0
Q: Auto mark answer for question which are not accepted?

saurabh kambleAccepting the answers of question which are old around 6 - 12 months old on following clause: Number of vote ups for that answer Number of edits and comments on answer Number of down vote answer to be excluded regardless of the total number of vote Comment of the person asking question (like ...

 
0
Q: How to show that det(A)≤1?

Sokserey Manenter image description here Should I use the definition of matrix: det(A)=Σ sgn( σ )a[1σ(1)]a[2σ(2)]...a[nσ(n)] ? I don't understand what is a[iσ(i)] ? Where i=1,2....n. Or is there another way to solve it?

0
Q: surjections/injection Discret Math

RonSuppose that f : A → B is a function. If S ⊆ A, then we define f(S) to be the set f(S) = {f(x) : x ∈ S}. (So for example, if f : R → R is given by f(x) = x^2 , then we have f({1, 2, 3}) = {1, 4, 9}, we have f({−2, 2, 3}) = {4, 9}, and f([−2, 2]) = [0, 4].) a) Prove that if S ⊆ A and T ⊆ A, then f...

 
6:09 AM
0
Q: Why is my question on martingales off-topic and not migrated?

BCLCHere is my question: Show that $E[X_T | T < \infty] \le E[X_0]$ and $cP(\sup X_n \ge c) \le E[X_0]$ I said in the beginning to pls migrate if needed. Could it be on-topic here if I made some edits? What should I do?

0
Q: Are programming questions on topic?

caseyI'm asking this because of the closing of this question asking how to plot wind barbs in python. The reception from the community seems to be split. One one hand it was upvoted to +7, and on the other hand it was closed. Even the reviews were a going both ways. Furthermore, we've been incon...

 
0
Q: $\forall v (d_v \ge \delta)\Rightarrow $ $G$ has tow cycle of length of $\delta+1$

AmirIf every degree's vertex of graph $G(V,E)$ bigger than $\delta$. Prove $G$ has two cycle of length $\delta+1$.

0
Q: Comass of a differential form

user135988In the wikipedia article on currents https://en.wikipedia.org/wiki/Current_%28mathematics%29 it is written that If $\omega$ is an m-form, then define its comass by $||\omega|| = \sup\{|\langle \omega,\xi\rangle| : \xi$ is a unit, simple m-vector$} I don't understand this definition. Can some...

0
Q: Seemingly Simple Intergal

Sat DIntegrate $f(x)$ from $0$ to $1$, such that $f(x) = \frac{x^2lnx}{sqrt(1-x^2)}$ I started off with the substitution $x=siny$, which resulted in the integrand reducing to $sin^2y.lnsiny$ Then I used the property of definite integrals that integral of $f(x)$ is the same as integral of $f(a+b-x)$...

 
6:37 AM
1
Q: Alignment issue in user profile badge section

Rahul NikateThere is a alignment issue in user profile badge section. Numbers for badge are not in one line. Here's the sample profile link for verification Here's the screen shot for this.

 
0
Q: how to calculate spectrum of a large collumn stochastic matrix

miosakiOkay, I have a collumn stochastic matrix of order $280\times 280$, the entries are given in an url in some webpage in row format. I need to find all the eigen values and eigen vector corresponding to the eigen value $1$ How can I proceed for that? What are the quick online tools are there?could ...

0
Q: Correlation between four (more) variables

tehseen fatimaIf A, B, C and D are four variables, the correlation matrix will be 4x4 matrix with elements, [1 Corr(A,B) Corr(A,C) Corr(A,D); Corr(B,A) 1 Corr(B,C) Corr(B,D); Corr(C,A) Corr(C,B) 1 Corr(C,D); Corr(D,A) Corr(D,B) Corr(D,C) 1] I have several questions: 1) What is the maximum possible valu...

Consider adding a tag for a broader subject area to which the question belongs. Some of these tags might fit. (autocomment)Normal Human 20 secs ago
0
Q: verifying minimum capacity of cut

curiousmind1995So I have a cut $(P,P')$ on some network and its capacity is 13. Now I'm told to assume that the current flow on the network is the max flow, is the cut of minimum capacity? So far all we've learned is that the max flow is at most equal to the capacity of the cut but I we haven't dealt with mini...

Questions tend to get more attention when they have a tag for a broad area of mathematics relevant to the question. Some of these tags might fit. (from a bot)Normal Human 20 secs ago
0
Q: In hyperbolic geometry, prove that parallel lines are not equidistant

Marissa FarinaIn Euclidean Geometry, parallel lines are equidistant. In hyperbolic geometry, it appears that parallel lines are $not$ equidistant. Is there a proof that supports this, or is it supposed to be trivial, or obvious because lines in hyperbolic geometry are/ can be curves. !https://graphics.stan...

 
6:56 AM
0
Q: List of questions disappearing

Tot ZamUnder the question menu, if I click on "unanswered" and then go back to "all questions" while on the "my tags" tab, all the questions disappear. It seems that the tag filter is not clearing. I need to refresh the whole question page to get the list of questions back.

 
0
Q: Proving a lower bound and upper bound?

AlbertI understand why the empty set is a lower bound and A is an upper bound. The only problem I am having is putting my thoughts into a mathematical solution. Can anyone help out? Thanks. Let A be a set and F ⊆ P(A) be a family of subsets of A. Prove that ∅ is a lower bound for (F;⊆) and that A is ...

Tagged proof-explanation. Proving a lower bound and upper bound?
0
Q: When is the tensor unit a generator?

X.G at MathLet $\mathcal{C}$ be a monoidal category with the tensor unit $I$. Then there is a "forgetful" functor from it to $Sets$: \begin{equation} \mathrm{Hom}(I,-). \end{equation} But in general, this functor is actually not a forgetful functor as it may not be faithful. My question is Is there a...

Questions tend to get more attention when they have a tag for a broad area of mathematics relevant to the question. Some of these tags might fit. (from a bot)Normal Human 21 secs ago
0
Q: Exact solution of $y$

smithShow that the exact solution of $$\epsilon \frac{d^2y}{dx^2}+\frac{dy}{dx}-2x=0$$ is $$y=(x-\epsilon)^2+\frac{1+2\epsilon}{\exp(-1/\epsilon)-1}\{\exp(-x/\epsilon)-1\}-\epsilon^2$$

Short title. Short question. Exact solution of $y$
 
0
Q: stackoverflow meta not showing in Mobile App

Jean-Pierre OosthuizenI am not able to see stackoverflow meta in on my Mobile App. Is there a reason for this?

 
7:18 AM
0
Q: The problem of estimating a probability distribution from a closely timed sample?

cqcn1991I have a dataset, which is the hourly mean wind speed of each day(24 points each day), for 20 years. And I'm planning for using this dataset to estimate the probability distribution of hourly mean wind speed The problem is that, clearly, the wind speed between consecutive hours are correlated, t...

0
Q: Equality between limit and integral with integrand whose diverges at some point.

userLet $f:[0,1]\times[0,1]\to\mathbb{R}\cup\{\pm\infty\}$ be a function such that, for some $\hat{x}\in(0,1)$, $\lim_{x\to\hat{x}}f(x,y)=\infty$ for each $y\in[0,1]$ and $f$ is continuous in $[0,\hat{x})$. Also let $x_{\varepsilon}\in[0,\hat{x})$ be a sequence such that $x_{\varepsilon}\to\hat{x}$ a...

0
Q: What is number of interaction between actors in social network?

Jamal HussainI was studying a definition in an article where author mentioned number of interactions between two actors in social networks. I am attaching the snapshot please help me out how to interpret it. Thanks enter image description here

0
Q: Partial Derivatives Concept Problem

LeungHere is the problem "Assume that the functions I :lR3 -> lR (F,g) : lR2 -+ lR are differentiable and they satisfies F(x1,x2) = I( x1, x2, g(x1,x2) ). Find the partial derivatives of F in terms of those of I and g" My attempts at the problem ; Since its the first time i e...

Title contains problem. Partial Derivatives Concept Problem
 
7:42 AM
0
Q: Prove divergence of sequence

t.kelso$a_n=1+n\sin\left(\frac{n\pi}{2}\right)$ So the above sequence is obviously not convergent because of the sine, but how could one prove this? Thanks for any pointers. Tom

Short title. Short question. Prove divergence of sequence
0
Q: determine convergence or divergence of a series, finding sum of series

marg_ocruzI am given the series $$\sum_{n=2}^\infty \frac{1+2^n}{3^n}$$ So I used ratio test for this one to see if this converges and I found out that it converges, however the answer key tells me there is a geometric sum for this and I don't know how to transform this geometrically because of the numera...

0
Q: Percentage of tonnage

eddieI need to create a spreadsheet outlining the percentage of the tonnage of a material vs. the number of sacks of drying agent it takes to solidify the material?

Short title. Short question. Percentage of tonnage
 
0
Q: Link does not parse correctly (dollar sign)

aeryaguzovYesterday, I gave an answer to the question on SO (Calling custom method before session auto destroy, already edited by community) In this question I posted a link to YII docs - http://www.yiiframework.com/doc-2.0/yii-web-session.html#$handler-detail On preview this link is parsed correctly, ...

 
8:02 AM
0
Q: Find cofficient for X^n for a Generating function

Argonne If given a GF F(x) = 1/(1-rx)^2, how do I find the coefficient for the term x^n? I can tell F(x) = A(x)^2, where A(x) is the GF for the sequence 1, r, r^2, r^3, r^4, .... but I don't know how to find the coefficient at x^n. Thanks.

This site uses MathJax formatting of formulas. More tips here. (autocomment)Normal Human 21 secs ago
 
hello
 
0
Q: Calculating double integral

YounesI need help to solve the following integral: \int_{0}^{x}\int_{0}^{y}\frac{s^{m}t^{n}}{‎\sqrt{‎‎(x-s)^2+(y-t)^2}}dtds‎ thank you

This site uses MathJax formatting of formulas. More tips here. (from a bot)Normal Human 21 secs ago
 
0
Q: I am a Mechanical Engineering student, can I be a theoretical physicist or a mathematician ? for hobby

Vinay5forPrimeI am a mechanical engineering student, but I want to hone my physics and mathematical skills other than that I will get in my engineering course. Is it possible to learn the skills required to be a hobby theoretical physicist or a mathematician. Can I be able to publish papers on maths or physic...

0
Q: Why I didn't get enough answers all the time?

user85361I posted a couple of questions in this site in the last year. I always post questions which I really involve with and had to answer in order to continue my research. But it's unfortunate that most of the time I didn't get the any attention or answer. I try to edit the answer, answer comments, eve...

2
Q: Syntax highlighting for ES2015 (ES6)

rink.attendant.6The syntax highlighting for JavaScript (lang-js) is not highlighting es6 code properly. For instance, unlike var, the let keyword is not highlighted. Arrow functions appear to be unrecognized and template strings are not highlighted at all let alone the placeholder expressions inside. Is this s...

 
8:29 AM
0
Q: Proof of subsets

SmallElephantSuppose $\Omega$ is bounded and that $1 \leq p \leq q \leq \infty $ . Prove the following statements: $L_q(\Omega) \subset L_p(\Omega)$ For $ k \in N^+ : W^k_q (\Omega) \subset W^k_p (\Omega)$ I have tried to used Holders ineq, for the first one, but doesnt seem to be working. If anybody has...

Short title. Proof of subsets
0
Q: How to notice symmetry?

mkropkowskiWhat is the easiest method to notice the symmetry of the following function without using any graphical tool: $$g(x)=\frac{1}{\pi \sqrt{4-x^2}}$$

Consider replacing (analysis) with a more specific tag for the relevant branch of analysis. (from a bot)Normal Human 21 secs ago
 
0
Q: Is there any way of discouraging people from editing incorrect code in questions?

BFDatabaseAdminQuite often, I'll see people suggesting edits that corrects code in questions. This is a fine idea in theory, except the error in the code may be the reason for the question, so should instead be an answer or a comment. Is there any way of discouraging, or at least warning about, this? E.g. http...

 
0
Q: Show that there exist a subsequence that converge uniformly to a infinitely differentiable function

LillyLet {fn} n ∈ N, be a sequence of infinitely differentiable functions (smooth function) on [a,b] such that for all integer k >= 0, there exist a real M_k such that |fn^(k) (x)| <= M_k for all x ∈ [a,b]. Show that there exist a subsequence that converges uniformly with all it's derivatives to a inf...

Consider replacing (analysis) with a more specific tag for the relevant branch of analysis. (autocomment)Normal Human 21 secs ago
0
Q: Choosing parametrization for complex integration with two branch cuts

Emir ŠemšićI am particularly interested in how Ron Gordon came up with the parametrization in his anser to this question: Inverse Laplace transform $\mathcal{L}^{-1}\left \{ \ln \left ( 1+\frac{w^{2}}{s^{2}}\right ) \right \}$

0
Q: Evaluate: $\int_{-\tfrac{1}{2}}^{\tfrac{1}{2}}\frac{\sin^4(nx)}{\sin^2(x)}$

measureprobEvaluate: $$\int_{-\tfrac{1}{2}}^{\tfrac{1}{2}}\frac{\sin^4(nx)}{\sin^2(x)}$$ Anyone able to provide a detailed prove of this? I can't seem to figure it out, I've tried using Euler's formula, but no luck.

0
Q: Longitudinal 1D wave equation Boundary Value Problem in an elastic rod

Lio2410I am trying to set up the 1D wave equation BVP for an elastic rod. Namely, an elastic rod of length L is fixed at x = 0, and is initially stretched by a length c, and so the end of the rod is at x = L + c. At time 0, it released. Also at time 0, velocity is approximately 0. The governing equation...

Questions tend to get more attention when they have a tag for a broad area of mathematics relevant to the question. Some of these tags might fit. (autocomment)Normal Human 20 secs ago
0
Q: Taking derivative of integral in application question

patrickhThe question reads as follows: A high-tech company purchases a new computing system whose initial value is $V$. The system will depreciate at the rate $f = f(t)$ and will accumulate maintenance costs at the rate $g = g(t)$, where $t$ is the time measure in months. The company wants to determine ...

Words such as question are uninformative in titles. Please edit the title so that it better describes the specifics of your question. Do not hesitate to make it longer or include a formula if needed. More tips here. (autocomment)Normal Human 20 secs ago
0
Q: Radial Plane Topology

AlgebryA subset of R2 is radially open if it contains an open line segment in each direction about each of its points. If A is a subset of the plane R2 and A intersects any line in at most 2 points, prove that R2/A is radially open. Ok, so I have to prove that in R2/A there is an open line segment i...

0
Q: Cloud cplex solve mixed integer semi-definite programming (MISDP)?

ArchyYuAs we know, cplex has the capabilities to solve LP, MIP, QCP, MIQCP, SOCP, and MISOCP. But, cloud cplex solve MISDP?

0
Q: Kuratowski Operations

User23I went through the concept of Kuratowski monoid in the paper by " B. J. Gardner and M. Jackson, The Kuratowski closure-complement theorem, New Zealand J. Math. 38 (2008), 9--44". They consider the set of all distinct operators on the topological space $( X,\tau )$ produced by composition of c...

0
Q: How stable are the results of various quadratic programming algorithms?

esarbeUsing 'coneprog' in R and 'cvxopt' in Python I'm getting slightly different results. Is that to be expected? How stable are the various algorithms compared to each other?

0
Q: please help me in this,I did it in MATLAB but give me wrong in (m) how i can do it?

user286513function r=isssdd(A,s) [m,n]= size(A); sbar=1:m; sbar(s)=[]; if m==n a=diag(abs(A)); B=abs(A-diag(diag(A))); rs= sum(B(:,s)')'; rsbar =sum(B(:,sbar)')'; c= (a(s)-rs(s))*(a(sbar)-rsbar(sbar))' > rsbar(s)*rs(sbar)'; if all(a(s)> rs(s)) & all(all(c-diag(diag(c))+eye(m))); ...

 
9:21 AM
-1
Q: Audit failed on an answer for a deleted question

Davide PastoreI just failed an audit. The problem is that the question wasn't there (it was deleted) and I only saw the answer that for my point of view would be ok. So now I have to wait for 2 days to review again. Is it ok?

 
0
Q: eFAST in sensitivity analysis

user295823I need to perform the parameter sensitivity analysis (SA) of the steady-states of my ODE model. Can I use the extended Fourier Amplitude Sensitivity Testing (eFAST) for this purpose? Should I use the time step long enough to reach the steady-state?

0
Q: Adapting poisson distribution to rare events

user3259937 Hi Everyone, This is my first post here. I have no knowledge of queuing theory, so forgive me if the question is quite basic. I have a poisson distribution that calculates the number of events at a given time interval. I have split the distribution into 5 minute time periods for a day, meanin...

Welcome to Math.SE, user3259937. Consider adding a tag for a broader subject area to which the question belongs. Some of these tags might fit. (from a bot)Normal Human 21 secs ago
0
Q: Taylor series estimation of differential equation

user295824I have a differential equation x'(t) = tx + t^4, with initial condition x(5)=3. I am asked to find the estimates using the taylor series method from o < t < 5 with h=0.01 steps. I get that you have to use the formula x1 = x0 + hx0' + h^2/2! x0'' .... but this is a recursion. If I was given initia...

0
Q: Versions of Tanaka's SDE

Robert W.Consider the following versions: $$dX_t=x_0+sgn(X_t)dW_t \tag1$$ $$dX_t=x_0+1_{(0,+\infty)}(X_t)dW_t \tag2$$ $$dX_t=x_0+1_{(-\infty,0]}(X_t)dW_t \tag3$$ SDE (1) is a classical example. SDE (2) has a strong solution $X_t=x_0+W_{t\wedge \sigma}$, where $\sigma=\inf\{t\ge 0 : x_0+W_t\le 0\}$. Ho...

Welcome to Math.SE, user295824. This site uses MathJax formatting of formulas. More tips here. (autocomment)Normal Human 21 secs ago
0
Q: Vector Proof involving Triangle

VectorProofN00bI'm stuck on the following homework question: Given the triangle $PQR$, with $X$ placed on $PR$ dividing it into a ratio of $2:3$, and $Y$ the midpoint of $PQ$, prove that if $Z$ is the midpoint of $QX$ and $YR$, then it divides $YR$ into a ratio of $1:3$. Note that this is to be solved pur...

Tagged proof-writing. Vector Proof involving Triangle
0
Q: Set of complex numbers

DaveIf I were to construct a certain set of complex numbers such that it is comprised of complex numbers whose norm is irrational, how can I show that this set is open? I was thinking about using the definition of an open set. I would show that each member of the set is an interior point. However I ...

0
Q: $(G=G{'}) \Rightarrow Z(G)= \{e \} $

AmirIf $G^{'}$ is Commutator subgroup of $G$ and $G=G{'} $. Can I show that $Z(G)= \{e \} $? I think its not True but i can not find example.

A title should not be all-MathJax; having some plain text helps with search and navigation. (autocomment)Normal Human 21 secs ago
 
9:58 AM
0
Q: please help,that is s-sdd matrices i did this program but some thing wrong i can not find it.

user286513function r=isssdd(A,s) [m,n]= size(A); sbar=1:m; sbar(s)=[]; if m==n a=diag(abs(A)); B=abs(A-diag(diag(A))); rs= sum(B(:,s)')'; rsbar =sum(B(:,sbar)')'; c= (a(s)-rs(s))*(a(sbar)-rsbar(sbar))' > rsbar(s)*rs(sbar)'; if all(a(s)> rs(s)) & all(all(c-diag(diag(c))+eye(m))); ...

0
Q: Is it correct: $d^2y = dxdy$

Тимофей ЛомоносовI have an ordinary differential equation in which I substitute $\xi = \xi(x), \eta = \eta(x,y)$ and I wish to express $y''_{xx}$ in terms of $\eta''_{\xi\xi}$. So I get an equation $$d^2\eta=\frac{\partial^2 \eta}{\partial x^2}+2\frac{\partial \eta}{\partial x\partial y}dxdy+\frac{\partial^2 \et...

Short title. Tagged differential-equations but mentions "partial". Is it correct: $d^2y = dxdy$
0
Q: Adding vector to span a space

gbox Let $A=\{\left(\begin{array}{c} 18 \\ 6 \\ -4 \\ 12 \end{array}\right),\left(\begin{array}{c} 6 \\ 2 \\ 2 \\ -6 \end{array}\right)\}$ find the vectos to be added so A will span $\mathbb{R}^4$? So what I did is : \begin{pmatrix} 18 & 6 & -4 & 12 \\ 6 & 2 & 2 & -6 \\ \end{pm...

0
Q: Concrete solution to the (oriented) Oberwolfach problem with one table

OblomovThe oriented Oberwolfach problem (with one table) and its solution are the following. In a meeting of $n$ people during $n-1$ days (combinatorists at Oberwolfach for concreteness), they all have diner around one table. As they only speak to their right neighbour, everybody wants to be seated eac...

0
Q: Chromatic number of a graph on a binary alphabet

Bertgiven the graph defined in this post: A binary sequence graph how would one go about finding its chromatic number?

0
Q: Quadrilaterals and figures

LalaAbc is a triangle in which l is the midpoint of ab and n is a point on AV such that an =2CN Aline thought l parallel to BN meets AC at M prove that AM=CN

Short title. Short question. Quadrilaterals and figures
 
10:22 AM
0
Q: Understanding of chain rule

jukka.aaltoMy book says that $f_U(u)=\Phi'(u)+\Phi'(-u)=\phi(u)+\phi(-u)$ because of the chain rule. Why is that?

Short title. Short question. Understanding of chain rule
0
Q: If $f(x)=\frac{\arcsin(1-\left\{x\right\})\times\arccos(1-\left\{x\right\})}{\sqrt{2\left\{x\right\}}\times(1-\left\{x\right\})}$

Vinod Kumar PuniaIf $f(x)=\frac{\arcsin(1-\left\{x\right\})\times\arccos(1-\left\{x\right\})}{\sqrt{2\left\{x\right\}}\times(1-\left\{x\right\})}$ Find $\lim_{x\to 0^+}f(x)$ and $\lim_{x\to 0^-}f(x)$.Where $\left\{x\right\} $is a fractional part function. As $x\to 0^+,\left\{x\right\}\to 0$,so $\lim_{x\to 0^+}...

0
Q: if f is continuous in a sequence of sets is it continuous in the union of them?

Arbër LlLet f be a continuous function on each sequence of compact sets {$K_i$}. Does it have to be continuous in the finite union of these sets $K= \bigcup\limits_{i=1}^n K_i $ ?

 
10:40 AM
0
Q: Time Complexity

Jakub KawalecPrepping for an exam and wondering whether I correctly calculated the time complexity. Function is given as: $function XYZ(n:integer)\\ begin for\ i:=1 \ do \ 2*n^2 \ do;\\ if \ n = 1\ then\ return (2);\\ else \ return(2*XYZ(\lfloor(XYZ(\lfloor n/2\rf...

Short title. Time Complexity
0
Q: Is this meters equivalent?

amir bahadoryLet $(X,d)$ be metric space and f be a one to one and continuous function, now we consider $ k(x,y)=d(f(x),f(y))$, is k(x,y) and d(x,y) equivalent ? In especial case we can consider $ k(x,y)= | \frac{x}{1+|x|} -\frac{y}{1+|y|} |$ and $d(x,y)=|x-y|$, now $f(x)=\frac{x}{1+|x|}$ and with definitio...

0
Q: Adjoint Operator with Matrix

ShalalehLet $\Omega \subset \Bbb R^n$ and $L$ be a linear differential operator 1 $$L:V\to W, V:=\mathcal C^1_0(\Omega;\Bbb R), W:=\mathcal C^0_0(\Omega; \Bbb R^{n*n})$$ $$L(u):=(\partial_j u_j - \partial_i u_j)_{i,j=1,...n}$$ equipped with the following inner products $$\lt u,v\gt_V:=\int_\Omega\sum_{i=...

 
0
Q: Push notifications for for Stack Overflow

Miguel GarciaThere is a fairly popular chrome extension called StackNotifier that let's you get notifications in chrome about answers to your questions, incoming questions etc. Chrome supports web push these days so perhaps it could be implemented natively on the site? That way it would work in Android as well.

0
Q: My issue was resolved, but not because the question itself was answered. What do I do with the question?

VegardI had a question on SO, the core of which was getting in my way of solving a particular problem. A commenter alerted me of an alternative way, which worked, but the alternative solution does not in any way answer the actual question. The question was "Why do I get this error using X?" and the cur...

 
0
Q: Variational formulation curl-div equation

avati91I want to prove that the following problem admits a unique solution $A_I\in H(curl;\Omega_I)\cap H(div;\Omega_I)$. $$ \begin{cases} curl(\varepsilon_I^{-1}curl A_I)=curl v_I\;\;\text{in}\,\,\Omega_I\\ div A_I=0\;\;\text{in}\,\,\Omega_I\\ A_I\cdot n_I=0\;\;\text{on}\,\,\Gamma\cup\partial\Omega\\ c...

Tagged pde, differential-equations. Tagged differential-equations but mentions "partial". Variational formulation curl-div equation
 
11:02 AM
0
Q: A group of monkeys are given two tasks to do.Probability

user290335can you help me in this question? I would like to know if I am doing it right. A group of monkeys are given two tasks to do, the second of which is harder than the first. A proportion (3/5) get the first one right and (7/15) get the second one right. If a monkey gets the first one right, it h...

This site uses MathJax formatting of formulas. More tips here. (from a bot)Normal Human 21 secs ago
 
11:13 AM
0
Q: Question regarding $\sum_{k=1}^{n}[k,n]^{(k,n)}$

user109899How to prove the following statement: If $n\in\mathbb{N}$ satisfies: $$ \left(\sum_{k=1}^{n}[k,n]^{(k,n)}, n^3\right)=1 $$ where $[k,n]$ and $(k,n)$ are the least common multiple resp. the greatest common divisor of $k$ and $n$, then $n$ is square-free. I started with tackling the sum: $$ \sum_...

Words such as question do not add information to titles. Please edit the title so that it better describes the specifics of your question. Do not hesitate to make it longer or include a formula if needed. More tips here. (autocomment)Normal Human 21 secs ago
0
Q: What is the isomorphism between the fields $(Z_2[x]^{<3},+_{x^3+x^2+1},\times_{x^3+x^2+1})$ and $(Z_2[x]^{<3},+_{x^3+x+1},\times_{x^3+x+1})$

GloomyThey are both Galois fields of order 8. I'm not exactly sure what the question means - how does one determine/describe an isomorphism?

Short question. [What is the isomorphism between the fields $(Z_2[x]^{<3},+_{x^3+x^2+1},\times_{x^3+x^2+1})$ and $(Z_2[x]^{<3},+_{x^3+x+1},\times_{x^3+x+1})$](math.stackexchange.com/q/1559599)
0
Q: why is 1 greater than 0 or 2 greater than 1

nime?? ¿¿????????????????????????!!!!!!!!!???????????????????????!!!!!!????? 0<1? 1<2?

 
0
Q: Horrible pixel error in Stack Overflow logo

jrturtonMy eyes, the goggles etc. Safari, OS X 10.11, main page.

 
11:47 AM
0
Q: (2x-y+4)dy+(x-2y+5dx)=0

SergeiHow to solve the following equation? (2x-y+4)dy+(x-2y+5dx)=0 It is necessary to determine the type and total solution.

Short title. Short question. (2x-y+4)dy+(x-2y+5dx)=0
0
Q: Transformation of variables to preserve geometric mean ranking

James LivermoreI have variables made of vectors of numbers that I need to transform. Calling these X Y Z, the arithmetic means of the numbers are M(X) Cheers

0
Q: Does there exist infinitely many $k$ such that $\pi(k)|k$.

Jack FrostLet $\pi(k)$ denote the number of primes $p$ such that $p\le k$. For $k=2,3,4,6,8,\cdots{},$ we have $\pi(k)|k$. Does there exist infinitely many integers $k$ such that $\pi(k)|k$ ?

 
12:16 PM
0
Q: Integral exists over the closure but not the set itself

DakvaIs there an open set $S$ with a bounded function $f$ such that $\int_\overline{S}f$ exists but $\int_Sf$ does not?

 
12:34 PM
0
Q: Big Theta with Negative Coefficient Problem

AlmondManI had a question in regards to solving a Big-Theta problem. Our professor wanted us to prove that $n^3 - 47n^2 + 18 = \Theta(n^3)$ and to do so rigorously, meaning he does not want us to use the below method: $\lim\limits_{n \to \infty} \dfrac{f(n)}{g(n)} = c$ $f(n) = \Theta(g(n))$ iff $0 < c <...

0
Q: Sign of real parts of an analytic function?

RamandiSuppose that $f(z)$ is an analytic function on complex plane. For all $z\in D_f$ We have $|f(z)^2-1|<1$. Prove That the sign of $Re f(z)$ is fixed for all $z$?

0
Q: Geometry problem solved with too advanced techniques?

Mr. Y A circle rests in the interior of the parabola with equation $y=x^2$ so that it is tangent to the parabola at two points. How much higher is the center of the circle than the points of tangency ? I've seen the solution of the problem here on Aops site and it left me a bit skeptical. I a...

 
0
Q: Writing a comment to Q/A in `Review | First Posts`

Tom BrunbergIn Review | First Posts (filtered with [delphi]) http://stackoverflow.com/review/first-posts/10441865 The question was tagged [delphi] although not related to Delphi. I wrote a comment to the question, asking 'How is this related to Delphi?' and at the moment of [Enter] to terminate the commen...

 
12:53 PM
0
Q: How to solve linear equation using inversion method?

M Abid MajeedI am not getting understand Inversion method to solve linear Equation 2x1 + 4x2 = 4 9x1 + 3x2 = 6 How to solve this please clarify steps.

0
Q: Binomial Distribution/probability

StuMA rise breeder is trying to cross yellow and red roses. He finds, on average, that 1 in every 4 of his test seeds produce a yellow rose. If he plants 12 seeds, determine the prob that: Exactly 2 roses are purely yellow. Less than 3 of the 12 are purely yellow. My attempt a: n=2 p=0.25 12C2= 66 6...

Consider adding a tag for a broader subject area to which the question belongs. Some of these tags might fit. (autocomment)Normal Human 21 secs ago
0
Q: Is $A^c$ and $B^c$ are homeomorphic?

mahdi moosazadehAssume that $A,B \subset R^2$ and $A,B$ are compact and connected is $A^c$ and $B^c$ are homeomorphic ? I don't have a good idea to solve that.

0
Q: Roots of ln of a square

Arc676Problem: $$ y=\ln(3x-2)^2 $$ State the domain and the coordinates of the point where the curve crosses the x-axis At first sight, you say that the domain is $x>\frac23$ because $\ln$ is undefined for negative numbers, so you just rearrange $3x-2>0$. But the input of $\ln$ is squared, whic...

0
Q: Proving the following limit statments

NotSure I need to prove those but i am really confused because i think i have a counter example for the first two : for the first : f(x) = {if x < 0 then 0. if 0 <= x < 4 then 1. if x >= 4 then x^3} in that way, if x -> 4, then the limit of f(x) is 64, but the limit of sqrt(4) = 2 is 1, and not sqrt(6...

This site uses MathJax formatting of formulas. More tips here. (autocomment)Normal Human 21 secs ago
0
Q: Matlab 'For' loop and line plotting [Beginners question]

Gauss function question2a(a,x) y=zeros(1000,1); y(1)=x; hold on; for j=2:1000 y(j) = a *y(j-1)*exp(-y(j-1)); plot(j, y); end hold off; end Is my code for a certain exercise, however I am not getting a line instead of a line I am getting a lot of dots. Can someon...

Title contains question. Tagged matlab. [Matlab 'For' loop and line plotting [Beginners question]](math.stackexchange.com/q/1559692)
 
1:30 PM
0
Q: What is wrong in my reputation calculation?

manetsusSee my reputation history first: So, the reputation calculation is: Cause Reputation Increase ------------------- ------------------- 1. Basic reputation 1 2. Sep 9 100 3. Oct 5 2 ...

 
0
Q: Weak Gateaux Derivative

IdonknowSuppose $X$ and $Y$ are Banach spaces. Denote a function $F:X \rightarrow Y$ and $U \subset X$ is an open set. Gateaux differential $F$ at $u \in U$ in the direction $\phi \in X$ is given by $$\lim_{t \rightarrow 0}\dfrac{F(u+t \phi) - F(u)}{t}.$$ In Wiki, there is this sentence in the sectio...

0
Q: Help with the proof that if $E$ is a finite extension of $F$, then ${E:F}$ divides $[E:F]$

H_HassanThe book proves the statement as follow: If $E$ is finite over $F$, then $E=F(\alpha_1,...,\alpha_n)$, where $\alpha_i \in \bar F$. Let $irr(\alpha_i,F(\alpha_1,..,\alpha_{i-1}))$ have $\alpha_i$ as one of $n_i$ distinct zeros that are all of a common multiplicity $v_i$, then $$[F(\alpha_1...

Title contains help. [Help with the proof that if $E$ is a finite extension of $F$, then ${E:F}$ divides $[E:F]$](math.stackexchange.com/q/1559708)
 
0
Q: Become a good Reviewer (Is there a Reference?)

swidmannHello I'm pretty new to SO. As you can see (my rep.) I'm pretty new the review stuff too. I've done a few review tasks, but I'm always catching myself while I'm clicking on the "SKIP" button, because I don't know how to handle this review task. I found a lot of helpful Answers here on meta, but...

 
0
Q: Is it possible to define a hankel transform for a function depending of a complex variable

Baptiste Hankel transform is defined by $F_{\nu}(k) = \int_0^{\infty}f(r)J_{\nu}(kr)rdr$, and the inverse transform by $f(r) = \int_0^{\infty}F_{\nu}(k)J_{\nu}(kr)kdk$, In my problem, r is a complex variable. Is it still possible to define the hankel transform by integrating on a complex path? Best...

Welcome to Math.SE, Baptiste . Questions tend to get more attention when they have a tag for a broad area of mathematics relevant to the question. Some of these tags might fit. (autocomment)Normal Human 21 secs ago
0
Q: invert a ''polynomial'' with poles (sounds ridiculous, but please refer to the details)

M. ZengI recently encountered some polynomial inversion in some physics literature, the simplified version would be the following: $$f(x)=\sum_{n=0}^{\infty} f_nx^n=\frac{g(x)}{(x-x_1)(x-x_2)}$$ where $g(x)$ is some power series in $x$. People may naturally think that $x_1$ and $x_2$ should just be two...

0
Q: Problem about Fourier coefficients in the Hilbert space $L^{2}([0,2Pi])$

jensLet f be a $2\pi$ periodic function. Assume that $f$ is quadratic integrable in the interval $[0,2\pi]$. Consider $f$ as a vector in the Hilbert space $L^{2}([0,2Pi])$. Give based on the Fourier coefficients of $f$ the best approximation of $f$ in $L^{2}([0,2Pi])$ as a linear combination of $sin(...

Title contains problem. [Problem about Fourier coefficients in the Hilbert space $L^{2}([0,2Pi])$](math.stackexchange.com/q/1559718)
0
Q: Augmented Algebras

Pryscilla SilvaRecently I started to study Operads. My reference is Algebraic Operads of Jean-Louis Loday and Bruno Vallette. In this book they define augmented algebra of the following form: an $\mathbb{K}$-algebra $A$ is augmented when there is a morphism of algebras $\epsilon: A\rightarrow \mathbb{K}$ called...

Short title. Augmented Algebras
 
2:00 PM
0
Q: Take an example of Integrate of root 2

user userI want to solve an example like this : $\int_{0}^{4}\sqrt{4^2-x^2}\ dx$ according to this equation :$$\int \sqrt{a^2-x^2}\ dx= \frac{1}{2}\left(x\sqrt{a^2-x^2}+a^2\sin^{-1}\left(\frac{x}{a}\right)\right)$$ I have problem with that sine . I want to see the solving step by step.

Title ends with a digit. Question contains step by step. Take an example of Integrate of root 2
0
Q: Is $r(^∗)=r^∗$ valid regular expression?

Mithlesh UpadhyayWhich of the following regular expression identities is/are TRUE? $r(^∗)=r^∗$ $(r^∗s^∗)=(r+s)^∗$ $(r+s)^∗=r^∗+s^∗$ $ r^∗s^∗=r^∗+s^∗$ AFAIK : I can't say anything, but it should be valid. $LHS\subset RHS$ $RHS\subset LHS$ $RHS\subset LHS$ My question is : Is $r(^∗)=r^∗$ valid regul...

0
Q: Integration expression problem

KennyYangI am trying to simplify an integration expression like this:$\int_0^{D}\frac{1}{x^{a}+1}dx$. I know that for a is 2, I can have it as arctan. But what about the general case? Thanks for your help!

Short title. Title contains problem. Integration expression problem
0
Q: How to access the values of RootOf within a Struct in Matlab

PaulI am solving a system of nonlinear equations in Matlab. The solution G returns a struct object and I can pull out the solution(s) x, y as S1=[G.x, G.y]. I cannot evaluate the values though. If I examine S1(1, 1) it contains a multiple of RootOf(cubic in a dummy variable z1) and if I try float(S1...

0
Q: Is the sum of transcendental and algebraic number transcendental number?

Ante PaladinI know almost nothing about transcendental numbers, I know the definition of them and maybe few results about them and that is all. But the question in the title somehow naturally arises when thinking about transcendental numbers. I think that it is okay to state it once more in the body of the...

Consider replacing (analysis) with a more specific tag for the relevant branch of analysis. (autocomment)Normal Human 20 secs ago
0
Q: Show a function is in $L_2$

RancSuppose $f\colon [0,\infty) \rightarrow \mathbb R$ is measurable, and suppose there exists $M>0$ s.t. $\forall h\in\mathbb L_2[0,\infty)$ We have $ |\int _{[0,\infty)}f(x)h(x)d\mu (x)|\leq M\cdot \|h\| _ {L_2}$. Show that $f\in L_2$. I got stuck at the end (My attempt is at the bottom). I would...

0
Q: Why is csc and sec defined as it is?

Jack FrostWhy is $\csc\theta=\frac{1}{\sin\theta}$ instead of $\csc\theta=\frac{1}{\cos\theta}$ and the same thing for $\sec\theta$? I wonder if there is an interesting reasoning for this.

0
Q: Ring $x^2=x$ then $2x=0$

Rishabh SareenIf in a ring $R$, $x^2=x$ for all $x$ then show that $2x=0$ and $x+y=0 \Rightarrow x=y.$ I am unable to proceed. Plz help.

Short title. Short question. Ring $x^2=x$ then $2x=0$
0
Q: Functional dependency: what does XY -> Z mean?

SingularityWhat does XY -> Z mean? Does it mean X -> Z and Y -> Z? Or does it just mean XY -> Z?

0
Q: Central limit theorem hard example!

sarahbenoistThe probability that a seed will germinate is 0.37. Suppose 124 seeds are planted. Use the Central Limit Theorem to determine the probability that at most 42 seeds germinate.

Title contains hard. Short question. Central limit theorem hard example!
0
Q: Two circles have an external tangent with length $36$cm .The shortest distance between these circles is $14$cm.

Vinod Kumar PuniaTwo circles have an external tangent with length $36$cm .The shortest distance between these circles is $14$cm.If the radius of the longer circle is $4$ times the radius of the smaller circle then find the radius of the larger circle. I dont know how to solve this problem.I tried but i failed....

0
Q: Prove that any countably incomplete ultraproduct of a collection of models is $\aleph_1$-saturated

Amir.H KianiI'm using this article for the proof. everything sounds well, but I don't think I have a proper comprehension on some (specially final) parts. For example, what is the function $f$ doing at the last stages and how is it working? please give me some intuition on what is going on in the proof.

0
Q: Proof formula of Proof by Contradiction $[ (\neg p \implies q) \land (\neg p\implies \neg q)] \implies p$

Educ I would like to proof the following statement : $$[ (\neg p \implies q) \land (\neg p\implies \neg q)] \implies p$$ My Proof: Methode $1$: Let $ R: [ (\neg p \implies q) \land (\neg p\implies \neg q)]$ Suppose that $R$ is true \begin{align} R &\iff [ (\neg p \implies q) \land (\ne...

Tagged proof-theory. [Proof formula of Proof by Contradiction $[ (\neg p \implies q) \land (\neg p\implies \neg q)] \implies p$](math.stackexchange.com/q/1559774)
0
Q: $f \in \mathbb R[x,y]$ be such that for some open set $U \subseteq \mathbb R^2$ , $f$ is zero in $U$ , then is $f$ identically $0$ in $\mathbb R^2$?

Saun DevLet $f \in \mathbb R[x,y]$ be such that for some open set $U \subseteq \mathbb R^2$ , $f(x,y)=0 , \forall (x,y) \in U$ ; then is it true that $f(x,y)=0 , \forall (x,y) \in \mathbb R^2$ ?

Short question. [$f \in \mathbb R[x,y]$ be such that for some open set $U \subseteq \mathbb R^2$ , $f$ is zero in $U$ , then is $f$ identically $0$ in $\mathbb R^2$?](math.stackexchange.com/q/1559782)
0
Q: How many natural numbers between 1 and 1000 which are divisible by 7 but not 2,3,5?

SRI HARSHAHow can we find the number of numbers between 1 and 1000 which are divisible by 7 but not 2,3,5?

0
Q: How to prove w^Tw\timesJw=0

gagarinwhere w is 3x1 matrix, J is symmetric matrix. I don't know how this equation is obtained..by mathematically. Please help me!!

Short title. Title ends with a digit. Short question. Question contains please. Tagged proof-writing. How to prove w^Tw\timesJw=0
 
2:56 PM
2
Q: Declined Flag on Question Where All Answers Are Bad

Sterling ArcherIs mule galaxy integrated into mule ESB? On this question, none of the answers really are suitable for SO. They scream comments. But my flag was declined. Why was it declined for an answer that says "but plugins exists" and doesn't give details?

0
Q: filter unanswered within a custom tab isn't filtering as it should

dippasI was testing the filters within the custom tab that I choose as one of my tabs to have and I noticed that the filter unanswered wasn't filtering as it should (at least the way I think) Because according to old navigation, the answered questions would be considered according to one of this point...

 
00:00 - 15:0015:00 - 00:00

« first day (32 days earlier)      last day (542 days later) »